use indicator function to prove independence











up vote
0
down vote

favorite












I would like to know why for $forall A in mathcal{F}$ $$E[1_{A}e^{itX}] = P(A)E[e^{itX}]$$ will imply the independence between $X$ and $mathcal{F}$. Thanks in advance!










share|cite|improve this question


























    up vote
    0
    down vote

    favorite












    I would like to know why for $forall A in mathcal{F}$ $$E[1_{A}e^{itX}] = P(A)E[e^{itX}]$$ will imply the independence between $X$ and $mathcal{F}$. Thanks in advance!










    share|cite|improve this question
























      up vote
      0
      down vote

      favorite









      up vote
      0
      down vote

      favorite











      I would like to know why for $forall A in mathcal{F}$ $$E[1_{A}e^{itX}] = P(A)E[e^{itX}]$$ will imply the independence between $X$ and $mathcal{F}$. Thanks in advance!










      share|cite|improve this question













      I would like to know why for $forall A in mathcal{F}$ $$E[1_{A}e^{itX}] = P(A)E[e^{itX}]$$ will imply the independence between $X$ and $mathcal{F}$. Thanks in advance!







      probability characteristic-functions






      share|cite|improve this question













      share|cite|improve this question











      share|cite|improve this question




      share|cite|improve this question










      asked yesterday









      VanDDF

      212




      212






















          3 Answers
          3






          active

          oldest

          votes

















          up vote
          1
          down vote













          Use simple function approximation to prove that $Ee^{itY}e^{itX}=Ee^{itY}Ee^{itX}$ for all $t$ for any $mathcal F$ measurable $Y$. This proves that $X$ and $Y$ are independent for any $mathcal F$ measurable $Y$. QED.






          share|cite|improve this answer




























            up vote
            0
            down vote













            Hint: let $Omega$ denote the probability space and $F$ the cdf of $X$. Since $$int_A exp itx dF(x)int_Omega dF(y)=int_A dF(x)int_Omegaexp ity dF(y)$$(where I've inserted a factor of $1$ on the left-hand side),$$int_Omega dF(y)int_A dF(x) [exp itx-exp ity]=0.$$






            share|cite|improve this answer




























              up vote
              0
              down vote













              Note that for $X= 1_A$, $Ain mathcal{F}$, it holds that$$E[e^{i(sX+tY)}] = E[(e^{is}1_A + 1_{A^c})e^{itY}]= (e^{is}P(A)+P(A^c))E[e^{itY}] = E[e^{isX}]E[e^{itY}].$$
              Thus $1_A$ and $Y$ are independent for all $Ainmathcal{F}$, and we conclude that $Y$ and $mathcal{F}$ are independent.






              share|cite|improve this answer





















                Your Answer





                StackExchange.ifUsing("editor", function () {
                return StackExchange.using("mathjaxEditing", function () {
                StackExchange.MarkdownEditor.creationCallbacks.add(function (editor, postfix) {
                StackExchange.mathjaxEditing.prepareWmdForMathJax(editor, postfix, [["$", "$"], ["\\(","\\)"]]);
                });
                });
                }, "mathjax-editing");

                StackExchange.ready(function() {
                var channelOptions = {
                tags: "".split(" "),
                id: "69"
                };
                initTagRenderer("".split(" "), "".split(" "), channelOptions);

                StackExchange.using("externalEditor", function() {
                // Have to fire editor after snippets, if snippets enabled
                if (StackExchange.settings.snippets.snippetsEnabled) {
                StackExchange.using("snippets", function() {
                createEditor();
                });
                }
                else {
                createEditor();
                }
                });

                function createEditor() {
                StackExchange.prepareEditor({
                heartbeatType: 'answer',
                convertImagesToLinks: true,
                noModals: true,
                showLowRepImageUploadWarning: true,
                reputationToPostImages: 10,
                bindNavPrevention: true,
                postfix: "",
                imageUploader: {
                brandingHtml: "Powered by u003ca class="icon-imgur-white" href="https://imgur.com/"u003eu003c/au003e",
                contentPolicyHtml: "User contributions licensed under u003ca href="https://creativecommons.org/licenses/by-sa/3.0/"u003ecc by-sa 3.0 with attribution requiredu003c/au003e u003ca href="https://stackoverflow.com/legal/content-policy"u003e(content policy)u003c/au003e",
                allowUrls: true
                },
                noCode: true, onDemand: true,
                discardSelector: ".discard-answer"
                ,immediatelyShowMarkdownHelp:true
                });


                }
                });














                 

                draft saved


                draft discarded


















                StackExchange.ready(
                function () {
                StackExchange.openid.initPostLogin('.new-post-login', 'https%3a%2f%2fmath.stackexchange.com%2fquestions%2f2996392%2fuse-indicator-function-to-prove-independence%23new-answer', 'question_page');
                }
                );

                Post as a guest
































                3 Answers
                3






                active

                oldest

                votes








                3 Answers
                3






                active

                oldest

                votes









                active

                oldest

                votes






                active

                oldest

                votes








                up vote
                1
                down vote













                Use simple function approximation to prove that $Ee^{itY}e^{itX}=Ee^{itY}Ee^{itX}$ for all $t$ for any $mathcal F$ measurable $Y$. This proves that $X$ and $Y$ are independent for any $mathcal F$ measurable $Y$. QED.






                share|cite|improve this answer

























                  up vote
                  1
                  down vote













                  Use simple function approximation to prove that $Ee^{itY}e^{itX}=Ee^{itY}Ee^{itX}$ for all $t$ for any $mathcal F$ measurable $Y$. This proves that $X$ and $Y$ are independent for any $mathcal F$ measurable $Y$. QED.






                  share|cite|improve this answer























                    up vote
                    1
                    down vote










                    up vote
                    1
                    down vote









                    Use simple function approximation to prove that $Ee^{itY}e^{itX}=Ee^{itY}Ee^{itX}$ for all $t$ for any $mathcal F$ measurable $Y$. This proves that $X$ and $Y$ are independent for any $mathcal F$ measurable $Y$. QED.






                    share|cite|improve this answer












                    Use simple function approximation to prove that $Ee^{itY}e^{itX}=Ee^{itY}Ee^{itX}$ for all $t$ for any $mathcal F$ measurable $Y$. This proves that $X$ and $Y$ are independent for any $mathcal F$ measurable $Y$. QED.







                    share|cite|improve this answer












                    share|cite|improve this answer



                    share|cite|improve this answer










                    answered yesterday









                    Kavi Rama Murthy

                    38.6k31747




                    38.6k31747






















                        up vote
                        0
                        down vote













                        Hint: let $Omega$ denote the probability space and $F$ the cdf of $X$. Since $$int_A exp itx dF(x)int_Omega dF(y)=int_A dF(x)int_Omegaexp ity dF(y)$$(where I've inserted a factor of $1$ on the left-hand side),$$int_Omega dF(y)int_A dF(x) [exp itx-exp ity]=0.$$






                        share|cite|improve this answer

























                          up vote
                          0
                          down vote













                          Hint: let $Omega$ denote the probability space and $F$ the cdf of $X$. Since $$int_A exp itx dF(x)int_Omega dF(y)=int_A dF(x)int_Omegaexp ity dF(y)$$(where I've inserted a factor of $1$ on the left-hand side),$$int_Omega dF(y)int_A dF(x) [exp itx-exp ity]=0.$$






                          share|cite|improve this answer























                            up vote
                            0
                            down vote










                            up vote
                            0
                            down vote









                            Hint: let $Omega$ denote the probability space and $F$ the cdf of $X$. Since $$int_A exp itx dF(x)int_Omega dF(y)=int_A dF(x)int_Omegaexp ity dF(y)$$(where I've inserted a factor of $1$ on the left-hand side),$$int_Omega dF(y)int_A dF(x) [exp itx-exp ity]=0.$$






                            share|cite|improve this answer












                            Hint: let $Omega$ denote the probability space and $F$ the cdf of $X$. Since $$int_A exp itx dF(x)int_Omega dF(y)=int_A dF(x)int_Omegaexp ity dF(y)$$(where I've inserted a factor of $1$ on the left-hand side),$$int_Omega dF(y)int_A dF(x) [exp itx-exp ity]=0.$$







                            share|cite|improve this answer












                            share|cite|improve this answer



                            share|cite|improve this answer










                            answered yesterday









                            J.G.

                            17.6k11829




                            17.6k11829






















                                up vote
                                0
                                down vote













                                Note that for $X= 1_A$, $Ain mathcal{F}$, it holds that$$E[e^{i(sX+tY)}] = E[(e^{is}1_A + 1_{A^c})e^{itY}]= (e^{is}P(A)+P(A^c))E[e^{itY}] = E[e^{isX}]E[e^{itY}].$$
                                Thus $1_A$ and $Y$ are independent for all $Ainmathcal{F}$, and we conclude that $Y$ and $mathcal{F}$ are independent.






                                share|cite|improve this answer

























                                  up vote
                                  0
                                  down vote













                                  Note that for $X= 1_A$, $Ain mathcal{F}$, it holds that$$E[e^{i(sX+tY)}] = E[(e^{is}1_A + 1_{A^c})e^{itY}]= (e^{is}P(A)+P(A^c))E[e^{itY}] = E[e^{isX}]E[e^{itY}].$$
                                  Thus $1_A$ and $Y$ are independent for all $Ainmathcal{F}$, and we conclude that $Y$ and $mathcal{F}$ are independent.






                                  share|cite|improve this answer























                                    up vote
                                    0
                                    down vote










                                    up vote
                                    0
                                    down vote









                                    Note that for $X= 1_A$, $Ain mathcal{F}$, it holds that$$E[e^{i(sX+tY)}] = E[(e^{is}1_A + 1_{A^c})e^{itY}]= (e^{is}P(A)+P(A^c))E[e^{itY}] = E[e^{isX}]E[e^{itY}].$$
                                    Thus $1_A$ and $Y$ are independent for all $Ainmathcal{F}$, and we conclude that $Y$ and $mathcal{F}$ are independent.






                                    share|cite|improve this answer












                                    Note that for $X= 1_A$, $Ain mathcal{F}$, it holds that$$E[e^{i(sX+tY)}] = E[(e^{is}1_A + 1_{A^c})e^{itY}]= (e^{is}P(A)+P(A^c))E[e^{itY}] = E[e^{isX}]E[e^{itY}].$$
                                    Thus $1_A$ and $Y$ are independent for all $Ainmathcal{F}$, and we conclude that $Y$ and $mathcal{F}$ are independent.







                                    share|cite|improve this answer












                                    share|cite|improve this answer



                                    share|cite|improve this answer










                                    answered yesterday









                                    Song

                                    3816




                                    3816






























                                         

                                        draft saved


                                        draft discarded



















































                                         


                                        draft saved


                                        draft discarded














                                        StackExchange.ready(
                                        function () {
                                        StackExchange.openid.initPostLogin('.new-post-login', 'https%3a%2f%2fmath.stackexchange.com%2fquestions%2f2996392%2fuse-indicator-function-to-prove-independence%23new-answer', 'question_page');
                                        }
                                        );

                                        Post as a guest




















































































                                        Popular posts from this blog

                                        How do I know what Microsoft account the skydrive app is syncing to?

                                        When does type information flow backwards in C++?

                                        Grease: Live!